Skip to content

Q4

← Back

Basic Info

Probability
└── Homework
    └── W5
        └── Q4.tex

Preview

\documentclass[12pt]{article}

\usepackage{graphicx}
\usepackage{amssymb}
\usepackage{amsmath}
\usepackage[margin=1in]{geometry}
\usepackage{fancyhdr}
\usepackage{enumerate}
\usepackage[shortlabels]{enumitem}

\pagestyle{fancy}
\fancyhead[l]{Li Yifeng}
\fancyhead[c]{Homework \#5}
\fancyhead[r]{\today}
\fancyfoot[c]{\thepage}
\renewcommand{\headrulewidth}{0.2pt}
\setlength{\headheight}{15pt}

\newcommand{\bP}{\mathbb{P}}

\begin{document}

    \section*{Question 4}

    \noindent We are given a bowl that contains $6$ balls, numbered from $1$ to $6$. We extract two balls and denote $X$ and $Y$ the numbers on the ball obtained at the first (second) extraction, and $W = \max(X,Y)$ the maximum value obtained. In all scenarios, describe the probability distribution of $W$.

    \bigskip

    \begin{enumerate}[start=1,label={\bfseries Part \arabic*:},leftmargin=0in]
        \bigskip\item In the first scenario, assume that the extractions are made with replacement.

        \subsection*{Solution}

            By applying the principle of symmetry, easy to define the probability space as

            \[
            \begin{aligned}
                \Omega &= \{1,2,\dots,6\}\\
                \mathcal{F} &= \mathcal{P}(\Omega)\\
                \bP &:\enspace \bP(\{1\}) = \bP(\{2\}) = \dots = \bP(\{6\}) = \frac{1}{6}
            \end{aligned}
            \]

            Then as given, we have

            \[
            \begin{aligned}
                X,Y &:\enspace \Omega \rightarrow R\\
                W &= \max(X,Y)\\
                R &= \{1,2,\dots,6\}
            \end{aligned}
            \]

            Then we have the probability distribution of $W$ in scenario \#1 (denoted as $W_1$ and $p_1$)

            \[
            p_1(w_1) =
            \begin{cases}
                \begin{aligned}
                    \bP(W_1 = 1) &= \bP(\{(1,1)\}) &= \frac{1}{6^2} = \frac{1}{36},&\quad w_1 = 1\\
                    \bP(W_1 = 2) &= \bP(\{(1,2),(2,1),(2,2)\}) &= \frac{3}{36},&\quad w_1 = 2\\
                    \bP(W_1 = 3) &= \bP(\{(1,3),(2,3),\dots,(3,3)\}) &= \frac{5}{36},&\quad w_1 = 3\\
                    \bP(W_1 = 4) &= \bP(\{(1,4),(2,4),\dots,(4,4)\}) &= \frac{7}{36},&\quad w_1 = 4\\
                    \bP(W_1 = 5) &= \bP(\{(1,5),(2,5),\dots,(5,5)\}) &= \frac{9}{36},&\quad w_1 = 5\\
                    \bP(W_1 = 6) &= \bP(\{(1,6),(2,6),\dots,(6,6)\}) &= \frac{11}{36},&\quad w_1 = 6
                \end{aligned}
            \end{cases}
            \]

            Or a general formula without cases

            \[p_1(w_1) = \frac{2w_1-1}{6^2} = \frac{2w_1-1}{36},\quad w_1\in \{1,2,\dots,6\}\]

        \subsection*{Answer}

            \[\boxed{p_1(w_1) =
                \begin{cases}
                    \begin{aligned}
                        \frac{1}{36},&\quad w_1 = 1\\
                        \frac{3}{36},&\quad w_1 = 2\\
                        \frac{5}{36},&\quad w_1 = 3\\
                        \frac{7}{36},&\quad w_1 = 4\\
                        \frac{9}{36},&\quad w_1 = 5\\
                        \frac{11}{36},&\quad w_1 = 6
                    \end{aligned}
            \end{cases}}\]

        \bigskip\item In the second scenario, assume that the extractions are performed without replacement.

        \subsection*{Solution}

            Easy to see that

            \[
            p_2(w_2) =
            \begin{cases}
                \begin{aligned}
                    \bP(W_2 = 2) &= \bP(\{(1,2),(2,1)\}) &= \frac{2}{6\times 5} = \frac{1}{15},&\quad w_2 = 2\\
                    \bP(W_2 = 3) &= \bP(\{(1,3),(2,3),\dots,(3,2)\}) &= \frac{2}{15},&\quad w_2 = 3\\
                    \bP(W_2 = 4) &= \bP(\{(1,4),(2,4),\dots,(4,3)\}) &= \frac{3}{15},&\quad w_2 = 4\\
                    \bP(W_2 = 5) &= \bP(\{(1,5),(2,5),\dots,(5,4)\}) &= \frac{4}{15},&\quad w_2 = 5\\
                    \bP(W_2 = 6) &= \bP(\{(1,6),(2,6),\dots,(6,5)\}) &= \frac{5}{15},&\quad w_2 = 6\\
                \end{aligned}
            \end{cases}
            \]

            Or a general formula without cases

            \[p_2(w_2) = \frac{2w_2-2}{6\times 5} = \frac{w_2-1}{15},\quad w_2\in \{2,3,\dots,6\}\]

        \subsection*{Answer}

            \[\boxed{p_2(w_2) =
                \begin{cases}
                    \begin{aligned}
                        \frac{1}{15},&\quad w_2 = 2\\
                        \frac{2}{15},&\quad w_2 = 3\\
                        \frac{3}{15},&\quad w_2 = 4\\
                        \frac{4}{15},&\quad w_2 = 5\\
                        \frac{5}{15},&\quad w_2 = 6\\
                    \end{aligned}
            \end{cases}}\]

        \bigskip\item In the third scenario, assume that after the first extraction, we replace the ball in the urn, together with another one with the same number.

        \subsection*{Solution}

            Easy to see that

            \[
            p_3(w_3) =
            \begin{cases}
                \begin{aligned}
                    \bP(W_3 = 1) &= \bP(\{(1,1)\}) &= \frac{1}{6}\times \frac{2}{7} &= \frac{1}{21},&\quad w_3 = 1\\
                    \bP(W_3 = 2) &= \bP(\{(1,2),(2,1),(2,2)\}) &= 2\left(\frac{1}{6}\times \frac{1}{7}\right) + \frac{2}{42} &= \frac{2}{21},&\quad w_3 = 2\\
                    \bP(W_3 = 3) &= \bP(\{(1,3),(2,3),\dots,(3,3)\}) &= 4\left(\frac{1}{42}\right) + \frac{2}{42} &= \frac{3}{21},&\quad w_3 = 3\\
                    \bP(W_3 = 4) &= \bP(\{(1,4),(2,4),\dots,(4,4)\}) &= 6\left(\frac{1}{42}\right) + \frac{2}{42} &= \frac{4}{21},&\quad w_3 = 4\\
                    \bP(W_3 = 5) &= \bP(\{(1,5),(2,5),\dots,(5,5)\}) &= 8\left(\frac{1}{42}\right) + \frac{2}{42} &= \frac{5}{21},&\quad w_3 = 5\\
                    \bP(W_3 = 6) &= \bP(\{(1,6),(2,6),\dots,(6,6)\}) &= 10\left(\frac{1}{42}\right) + \frac{2}{42} &= \frac{6}{21},&\quad w_3 = 6
                \end{aligned}
            \end{cases}
            \]

            Or a general formula without cases

            \[p_3(w_3) = \frac{w_3}{21},\quad w_3\in \{1,2,\dots,6\}\]

        \subsection*{Answer}

            \[\boxed{p_3(w_3) =
                \begin{cases}
                    \begin{aligned}
                        \frac{1}{21},&\quad w_3 = 1\\
                        \frac{2}{21},&\quad w_3 = 2\\
                        \frac{3}{21},&\quad w_3 = 3\\
                        \frac{4}{21},&\quad w_3 = 4\\
                        \frac{5}{21},&\quad w_3 = 5\\
                        \frac{6}{21},&\quad w_3 = 6
                    \end{aligned}
            \end{cases}}\]
    \end{enumerate}

\end{document}